Difference between revisions of "1997 AHSME Problems/Problem 14"

Line 4: Line 4:
  
 
<math> \textbf{(A)}\ 81\qquad\textbf{(B)}\ 84\qquad\textbf{(C)}\ 87\qquad\textbf{(D)}\ 90\qquad\textbf{(E)}\ 102 </math>
 
<math> \textbf{(A)}\ 81\qquad\textbf{(B)}\ 84\qquad\textbf{(C)}\ 87\qquad\textbf{(D)}\ 90\qquad\textbf{(E)}\ 102 </math>
 +
 +
==Solution==
 +
 +
Let <math>x</math> be the population in <math>1996</math>, and let <math>k</math> be the constant of proportionality.
 +
 +
If <math>n=1994</math>, then the difference in population between <math>1996</math> and <math>1994</math> is directly proportional to the population in <math>1995</math>.
 +
 +
Translating this sentence, <math>(x - 39) = k(60)</math>
 +
 +
Similarly, letting <math>n=1995</math> gives the sentence <math>(123 - 60) = kx</math>
 +
 +
Since <math>kx = 63</math>, we have <math>k = \frac{63}{x}</math>
 +
 +
Plugging this into the first equation, we have:
 +
 +
<math>(x - 39) = \frac{60\cdot 63}{x}</math>
 +
 +
<math>x - 39 = \frac{3780}{x}</math>
 +
 +
<math>x^2 - 39x - 3780 = 0</math>
 +
 +
<math>(x - 84)(x + 45) = 0</math>
 +
 +
Since <math>x>0</math>, we must have <math>x=84</math>, and the answer is <math>\boxed{B}</math>.
  
 
== See also ==
 
== See also ==
 
{{AHSME box|year=1997|num-b=13|num-a=15}}
 
{{AHSME box|year=1997|num-b=13|num-a=15}}

Revision as of 11:03, 9 August 2011

Problem

The number of geese in a flock increases so that the difference between the populations in year $n+2$ and year $n$ is directly proportional to the population in year $n+1$. If the populations in the years $1994$, $1995$, and $1997$ were $39$, $60$, and $123$, respectively, then the population in $1996$ was

$\textbf{(A)}\ 81\qquad\textbf{(B)}\ 84\qquad\textbf{(C)}\ 87\qquad\textbf{(D)}\ 90\qquad\textbf{(E)}\ 102$

Solution

Let $x$ be the population in $1996$, and let $k$ be the constant of proportionality.

If $n=1994$, then the difference in population between $1996$ and $1994$ is directly proportional to the population in $1995$.

Translating this sentence, $(x - 39) = k(60)$

Similarly, letting $n=1995$ gives the sentence $(123 - 60) = kx$

Since $kx = 63$, we have $k = \frac{63}{x}$

Plugging this into the first equation, we have:

$(x - 39) = \frac{60\cdot 63}{x}$

$x - 39 = \frac{3780}{x}$

$x^2 - 39x - 3780 = 0$

$(x - 84)(x + 45) = 0$

Since $x>0$, we must have $x=84$, and the answer is $\boxed{B}$.

See also

1997 AHSME (ProblemsAnswer KeyResources)
Preceded by
Problem 13
Followed by
Problem 15
1 2 3 4 5 6 7 8 9 10 11 12 13 14 15 16 17 18 19 20 21 22 23 24 25 26 27 28 29 30
All AHSME Problems and Solutions